Finding the maximum displacement of a bob from its equilibrium position using algebraic/trigonometric methods












0














The velocity of the bob is



$$v(t)=3sin{t}+4cos{t}$$



At $t=0$, the bob is $1$ unit from the equilibrium position.



I set the derivative equal to zero to find the max displacement, but I got a negative answer ($tapprox-0.927$). $t$ has to be greater than zero, but I don't know how to find the appropriate answer given the one I have.










share|cite|improve this question




















  • 2




    It asks you to find the maximum DISPLACEMENT, which is the integral of $v(t)$. So you need to find the integral first.
    – KittyL
    Mar 4 '15 at 0:13










  • That makes sense, and maybe I explained it wrong but those were the instructions I followed on the problem
    – user220799
    Mar 4 '15 at 0:31










  • Is it rectilinear motion or circular? How is equilibrium defined? ( v>0? \omega =0 ). In the latter case what is the string length and position of point of suspensoin to which the bob is connected? Is the angular velocity 1 radian per second?
    – Narasimham
    May 20 '18 at 17:34


















0














The velocity of the bob is



$$v(t)=3sin{t}+4cos{t}$$



At $t=0$, the bob is $1$ unit from the equilibrium position.



I set the derivative equal to zero to find the max displacement, but I got a negative answer ($tapprox-0.927$). $t$ has to be greater than zero, but I don't know how to find the appropriate answer given the one I have.










share|cite|improve this question




















  • 2




    It asks you to find the maximum DISPLACEMENT, which is the integral of $v(t)$. So you need to find the integral first.
    – KittyL
    Mar 4 '15 at 0:13










  • That makes sense, and maybe I explained it wrong but those were the instructions I followed on the problem
    – user220799
    Mar 4 '15 at 0:31










  • Is it rectilinear motion or circular? How is equilibrium defined? ( v>0? \omega =0 ). In the latter case what is the string length and position of point of suspensoin to which the bob is connected? Is the angular velocity 1 radian per second?
    – Narasimham
    May 20 '18 at 17:34
















0












0








0







The velocity of the bob is



$$v(t)=3sin{t}+4cos{t}$$



At $t=0$, the bob is $1$ unit from the equilibrium position.



I set the derivative equal to zero to find the max displacement, but I got a negative answer ($tapprox-0.927$). $t$ has to be greater than zero, but I don't know how to find the appropriate answer given the one I have.










share|cite|improve this question















The velocity of the bob is



$$v(t)=3sin{t}+4cos{t}$$



At $t=0$, the bob is $1$ unit from the equilibrium position.



I set the derivative equal to zero to find the max displacement, but I got a negative answer ($tapprox-0.927$). $t$ has to be greater than zero, but I don't know how to find the appropriate answer given the one I have.







calculus trigonometry






share|cite|improve this question















share|cite|improve this question













share|cite|improve this question




share|cite|improve this question








edited Apr 16 '17 at 15:38









Rodrigo de Azevedo

12.8k41855




12.8k41855










asked Mar 3 '15 at 23:59









user220799

11




11








  • 2




    It asks you to find the maximum DISPLACEMENT, which is the integral of $v(t)$. So you need to find the integral first.
    – KittyL
    Mar 4 '15 at 0:13










  • That makes sense, and maybe I explained it wrong but those were the instructions I followed on the problem
    – user220799
    Mar 4 '15 at 0:31










  • Is it rectilinear motion or circular? How is equilibrium defined? ( v>0? \omega =0 ). In the latter case what is the string length and position of point of suspensoin to which the bob is connected? Is the angular velocity 1 radian per second?
    – Narasimham
    May 20 '18 at 17:34
















  • 2




    It asks you to find the maximum DISPLACEMENT, which is the integral of $v(t)$. So you need to find the integral first.
    – KittyL
    Mar 4 '15 at 0:13










  • That makes sense, and maybe I explained it wrong but those were the instructions I followed on the problem
    – user220799
    Mar 4 '15 at 0:31










  • Is it rectilinear motion or circular? How is equilibrium defined? ( v>0? \omega =0 ). In the latter case what is the string length and position of point of suspensoin to which the bob is connected? Is the angular velocity 1 radian per second?
    – Narasimham
    May 20 '18 at 17:34










2




2




It asks you to find the maximum DISPLACEMENT, which is the integral of $v(t)$. So you need to find the integral first.
– KittyL
Mar 4 '15 at 0:13




It asks you to find the maximum DISPLACEMENT, which is the integral of $v(t)$. So you need to find the integral first.
– KittyL
Mar 4 '15 at 0:13












That makes sense, and maybe I explained it wrong but those were the instructions I followed on the problem
– user220799
Mar 4 '15 at 0:31




That makes sense, and maybe I explained it wrong but those were the instructions I followed on the problem
– user220799
Mar 4 '15 at 0:31












Is it rectilinear motion or circular? How is equilibrium defined? ( v>0? \omega =0 ). In the latter case what is the string length and position of point of suspensoin to which the bob is connected? Is the angular velocity 1 radian per second?
– Narasimham
May 20 '18 at 17:34






Is it rectilinear motion or circular? How is equilibrium defined? ( v>0? \omega =0 ). In the latter case what is the string length and position of point of suspensoin to which the bob is connected? Is the angular velocity 1 radian per second?
– Narasimham
May 20 '18 at 17:34












2 Answers
2






active

oldest

votes


















0














I am not sure what are the instructions of the problem. I think the displacement can be found by integral, then set the derivative of the displacement to 0 to obtain the maximum. On the other hand, the derivative of the displacement is the velocity itself.



So setting the velocity to 0 would give you the time when the displacement is maximum.



$$3sin{t}+4cos{t}=0 Rightarrow frac{3}{5}sin{t}+frac{4}{5}cos{t}=0\
sin{(t+arccos{(3/5)})}=0$$



So $t=pi-arccos{(3/5)}$. Notice that it is periodic, so using $pi-$ or $2pi-$ are both OK.



Plugging this $t$ into your displacement function would give you the maximum.



An alternative way:



The displacement function is $-3cos{t}+4sin{t}+4$ by integral.



Using the same argument as above this gives you



$$5(-frac{3}{5}cos{t}+frac{4}{5}sin{t})+4$$



The quantity in parenthesis is $sin{(t+theta)}$ for some $theta$. Its maximum must be $1$. So the maximum of the displacement must be $9$.






share|cite|improve this answer





























    0














    Integrating and noting initial condition we have $x(t)=-3cos(t)+4 sin (t)+4$. This can be written as $x(t)=5cos(t-(pi-arctan(frac{4}{3}))+4$.



    Proof:



    Consider the vector $vec v=<-3,4>$ and the unit vector $vec w=<cos (theta), sin (theta)>$.



    $$vec v cdot vec w=|vec v||vec w| cos( text{angle, between, them})$$



    $$=(sqrt{3^2+4^2})(1) cos ((pi-arctan(frac{4}{3}))-theta)$$



    $$=(sqrt{3^2+4^2})(1) cos (theta-(pi-arctan(frac{4}{3})))$$






    share|cite|improve this answer























      Your Answer





      StackExchange.ifUsing("editor", function () {
      return StackExchange.using("mathjaxEditing", function () {
      StackExchange.MarkdownEditor.creationCallbacks.add(function (editor, postfix) {
      StackExchange.mathjaxEditing.prepareWmdForMathJax(editor, postfix, [["$", "$"], ["\\(","\\)"]]);
      });
      });
      }, "mathjax-editing");

      StackExchange.ready(function() {
      var channelOptions = {
      tags: "".split(" "),
      id: "69"
      };
      initTagRenderer("".split(" "), "".split(" "), channelOptions);

      StackExchange.using("externalEditor", function() {
      // Have to fire editor after snippets, if snippets enabled
      if (StackExchange.settings.snippets.snippetsEnabled) {
      StackExchange.using("snippets", function() {
      createEditor();
      });
      }
      else {
      createEditor();
      }
      });

      function createEditor() {
      StackExchange.prepareEditor({
      heartbeatType: 'answer',
      autoActivateHeartbeat: false,
      convertImagesToLinks: true,
      noModals: true,
      showLowRepImageUploadWarning: true,
      reputationToPostImages: 10,
      bindNavPrevention: true,
      postfix: "",
      imageUploader: {
      brandingHtml: "Powered by u003ca class="icon-imgur-white" href="https://imgur.com/"u003eu003c/au003e",
      contentPolicyHtml: "User contributions licensed under u003ca href="https://creativecommons.org/licenses/by-sa/3.0/"u003ecc by-sa 3.0 with attribution requiredu003c/au003e u003ca href="https://stackoverflow.com/legal/content-policy"u003e(content policy)u003c/au003e",
      allowUrls: true
      },
      noCode: true, onDemand: true,
      discardSelector: ".discard-answer"
      ,immediatelyShowMarkdownHelp:true
      });


      }
      });














      draft saved

      draft discarded


















      StackExchange.ready(
      function () {
      StackExchange.openid.initPostLogin('.new-post-login', 'https%3a%2f%2fmath.stackexchange.com%2fquestions%2f1174360%2ffinding-the-maximum-displacement-of-a-bob-from-its-equilibrium-position-using-al%23new-answer', 'question_page');
      }
      );

      Post as a guest















      Required, but never shown

























      2 Answers
      2






      active

      oldest

      votes








      2 Answers
      2






      active

      oldest

      votes









      active

      oldest

      votes






      active

      oldest

      votes









      0














      I am not sure what are the instructions of the problem. I think the displacement can be found by integral, then set the derivative of the displacement to 0 to obtain the maximum. On the other hand, the derivative of the displacement is the velocity itself.



      So setting the velocity to 0 would give you the time when the displacement is maximum.



      $$3sin{t}+4cos{t}=0 Rightarrow frac{3}{5}sin{t}+frac{4}{5}cos{t}=0\
      sin{(t+arccos{(3/5)})}=0$$



      So $t=pi-arccos{(3/5)}$. Notice that it is periodic, so using $pi-$ or $2pi-$ are both OK.



      Plugging this $t$ into your displacement function would give you the maximum.



      An alternative way:



      The displacement function is $-3cos{t}+4sin{t}+4$ by integral.



      Using the same argument as above this gives you



      $$5(-frac{3}{5}cos{t}+frac{4}{5}sin{t})+4$$



      The quantity in parenthesis is $sin{(t+theta)}$ for some $theta$. Its maximum must be $1$. So the maximum of the displacement must be $9$.






      share|cite|improve this answer


























        0














        I am not sure what are the instructions of the problem. I think the displacement can be found by integral, then set the derivative of the displacement to 0 to obtain the maximum. On the other hand, the derivative of the displacement is the velocity itself.



        So setting the velocity to 0 would give you the time when the displacement is maximum.



        $$3sin{t}+4cos{t}=0 Rightarrow frac{3}{5}sin{t}+frac{4}{5}cos{t}=0\
        sin{(t+arccos{(3/5)})}=0$$



        So $t=pi-arccos{(3/5)}$. Notice that it is periodic, so using $pi-$ or $2pi-$ are both OK.



        Plugging this $t$ into your displacement function would give you the maximum.



        An alternative way:



        The displacement function is $-3cos{t}+4sin{t}+4$ by integral.



        Using the same argument as above this gives you



        $$5(-frac{3}{5}cos{t}+frac{4}{5}sin{t})+4$$



        The quantity in parenthesis is $sin{(t+theta)}$ for some $theta$. Its maximum must be $1$. So the maximum of the displacement must be $9$.






        share|cite|improve this answer
























          0












          0








          0






          I am not sure what are the instructions of the problem. I think the displacement can be found by integral, then set the derivative of the displacement to 0 to obtain the maximum. On the other hand, the derivative of the displacement is the velocity itself.



          So setting the velocity to 0 would give you the time when the displacement is maximum.



          $$3sin{t}+4cos{t}=0 Rightarrow frac{3}{5}sin{t}+frac{4}{5}cos{t}=0\
          sin{(t+arccos{(3/5)})}=0$$



          So $t=pi-arccos{(3/5)}$. Notice that it is periodic, so using $pi-$ or $2pi-$ are both OK.



          Plugging this $t$ into your displacement function would give you the maximum.



          An alternative way:



          The displacement function is $-3cos{t}+4sin{t}+4$ by integral.



          Using the same argument as above this gives you



          $$5(-frac{3}{5}cos{t}+frac{4}{5}sin{t})+4$$



          The quantity in parenthesis is $sin{(t+theta)}$ for some $theta$. Its maximum must be $1$. So the maximum of the displacement must be $9$.






          share|cite|improve this answer












          I am not sure what are the instructions of the problem. I think the displacement can be found by integral, then set the derivative of the displacement to 0 to obtain the maximum. On the other hand, the derivative of the displacement is the velocity itself.



          So setting the velocity to 0 would give you the time when the displacement is maximum.



          $$3sin{t}+4cos{t}=0 Rightarrow frac{3}{5}sin{t}+frac{4}{5}cos{t}=0\
          sin{(t+arccos{(3/5)})}=0$$



          So $t=pi-arccos{(3/5)}$. Notice that it is periodic, so using $pi-$ or $2pi-$ are both OK.



          Plugging this $t$ into your displacement function would give you the maximum.



          An alternative way:



          The displacement function is $-3cos{t}+4sin{t}+4$ by integral.



          Using the same argument as above this gives you



          $$5(-frac{3}{5}cos{t}+frac{4}{5}sin{t})+4$$



          The quantity in parenthesis is $sin{(t+theta)}$ for some $theta$. Its maximum must be $1$. So the maximum of the displacement must be $9$.







          share|cite|improve this answer












          share|cite|improve this answer



          share|cite|improve this answer










          answered Mar 4 '15 at 10:37









          KittyL

          13.8k31434




          13.8k31434























              0














              Integrating and noting initial condition we have $x(t)=-3cos(t)+4 sin (t)+4$. This can be written as $x(t)=5cos(t-(pi-arctan(frac{4}{3}))+4$.



              Proof:



              Consider the vector $vec v=<-3,4>$ and the unit vector $vec w=<cos (theta), sin (theta)>$.



              $$vec v cdot vec w=|vec v||vec w| cos( text{angle, between, them})$$



              $$=(sqrt{3^2+4^2})(1) cos ((pi-arctan(frac{4}{3}))-theta)$$



              $$=(sqrt{3^2+4^2})(1) cos (theta-(pi-arctan(frac{4}{3})))$$






              share|cite|improve this answer




























                0














                Integrating and noting initial condition we have $x(t)=-3cos(t)+4 sin (t)+4$. This can be written as $x(t)=5cos(t-(pi-arctan(frac{4}{3}))+4$.



                Proof:



                Consider the vector $vec v=<-3,4>$ and the unit vector $vec w=<cos (theta), sin (theta)>$.



                $$vec v cdot vec w=|vec v||vec w| cos( text{angle, between, them})$$



                $$=(sqrt{3^2+4^2})(1) cos ((pi-arctan(frac{4}{3}))-theta)$$



                $$=(sqrt{3^2+4^2})(1) cos (theta-(pi-arctan(frac{4}{3})))$$






                share|cite|improve this answer


























                  0












                  0








                  0






                  Integrating and noting initial condition we have $x(t)=-3cos(t)+4 sin (t)+4$. This can be written as $x(t)=5cos(t-(pi-arctan(frac{4}{3}))+4$.



                  Proof:



                  Consider the vector $vec v=<-3,4>$ and the unit vector $vec w=<cos (theta), sin (theta)>$.



                  $$vec v cdot vec w=|vec v||vec w| cos( text{angle, between, them})$$



                  $$=(sqrt{3^2+4^2})(1) cos ((pi-arctan(frac{4}{3}))-theta)$$



                  $$=(sqrt{3^2+4^2})(1) cos (theta-(pi-arctan(frac{4}{3})))$$






                  share|cite|improve this answer














                  Integrating and noting initial condition we have $x(t)=-3cos(t)+4 sin (t)+4$. This can be written as $x(t)=5cos(t-(pi-arctan(frac{4}{3}))+4$.



                  Proof:



                  Consider the vector $vec v=<-3,4>$ and the unit vector $vec w=<cos (theta), sin (theta)>$.



                  $$vec v cdot vec w=|vec v||vec w| cos( text{angle, between, them})$$



                  $$=(sqrt{3^2+4^2})(1) cos ((pi-arctan(frac{4}{3}))-theta)$$



                  $$=(sqrt{3^2+4^2})(1) cos (theta-(pi-arctan(frac{4}{3})))$$







                  share|cite|improve this answer














                  share|cite|improve this answer



                  share|cite|improve this answer








                  edited Jan 3 '17 at 1:17

























                  answered Jan 3 '17 at 0:54









                  Ahmed S. Attaalla

                  14.8k12049




                  14.8k12049






























                      draft saved

                      draft discarded




















































                      Thanks for contributing an answer to Mathematics Stack Exchange!


                      • Please be sure to answer the question. Provide details and share your research!

                      But avoid



                      • Asking for help, clarification, or responding to other answers.

                      • Making statements based on opinion; back them up with references or personal experience.


                      Use MathJax to format equations. MathJax reference.


                      To learn more, see our tips on writing great answers.





                      Some of your past answers have not been well-received, and you're in danger of being blocked from answering.


                      Please pay close attention to the following guidance:


                      • Please be sure to answer the question. Provide details and share your research!

                      But avoid



                      • Asking for help, clarification, or responding to other answers.

                      • Making statements based on opinion; back them up with references or personal experience.


                      To learn more, see our tips on writing great answers.




                      draft saved


                      draft discarded














                      StackExchange.ready(
                      function () {
                      StackExchange.openid.initPostLogin('.new-post-login', 'https%3a%2f%2fmath.stackexchange.com%2fquestions%2f1174360%2ffinding-the-maximum-displacement-of-a-bob-from-its-equilibrium-position-using-al%23new-answer', 'question_page');
                      }
                      );

                      Post as a guest















                      Required, but never shown





















































                      Required, but never shown














                      Required, but never shown












                      Required, but never shown







                      Required, but never shown

































                      Required, but never shown














                      Required, but never shown












                      Required, but never shown







                      Required, but never shown







                      Popular posts from this blog

                      Can a sorcerer learn a 5th-level spell early by creating spell slots using the Font of Magic feature?

                      Does disintegrating a polymorphed enemy still kill it after the 2018 errata?

                      A Topological Invariant for $pi_3(U(n))$